CAT Exam  >  CAT Tests  >  CAT Mock Test Series 2024  >  CAT Mock Test - 17 - CAT MCQ

CAT Mock Test - 17 - CAT MCQ


Test Description

30 Questions MCQ Test CAT Mock Test Series 2024 - CAT Mock Test - 17

CAT Mock Test - 17 for CAT 2024 is part of CAT Mock Test Series 2024 preparation. The CAT Mock Test - 17 questions and answers have been prepared according to the CAT exam syllabus.The CAT Mock Test - 17 MCQs are made for CAT 2024 Exam. Find important definitions, questions, notes, meanings, examples, exercises, MCQs and online tests for CAT Mock Test - 17 below.
Solutions of CAT Mock Test - 17 questions in English are available as part of our CAT Mock Test Series 2024 for CAT & CAT Mock Test - 17 solutions in Hindi for CAT Mock Test Series 2024 course. Download more important topics, notes, lectures and mock test series for CAT Exam by signing up for free. Attempt CAT Mock Test - 17 | 66 questions in 120 minutes | Mock test for CAT preparation | Free important questions MCQ to study CAT Mock Test Series 2024 for CAT Exam | Download free PDF with solutions
CAT Mock Test - 17 - Question 1

Directions: Read the following passage carefully and answer the questions that follow.

Bill Gates is a lot luckier than you might realise. He may be a very talented man who worked his way up from geek to the top spot on the list of the world's richest people. But his extreme success perhaps tells us more about the importance of circumstances beyond his control than it does about how skill and perseverance are rewarded.

We often fall for the idea that the exceptional performers are the most skilled or talented. But this is flawed. Exceptional performances tend to occur in exceptional circumstances. Top performers are often the luckiest people, who have benefited from being at the right place and right time. They are what we call outliers, whose performances may be examples set apart from the system that everyone else works within.

Many treat Gates, and other highly successful people like him, as deserving of huge attention and reward, as people from whom we could learn a lot about how to succeed. But assuming life's "winners" got there from performance alone is likely to lead to disappointment. Even if you could imitate everything Gates did, you would not be able to replicate his initial good fortune.

For example, Gates's upper-class background and private education enabled him to gain extra programming experience when less than 0.01% of his generation then had access to computers. His mother's social connection with IBM's chairman enabled him to gain a contract from the then-leading PC company that was crucial for establishing his software empire.

This is important because most customers who used IBM computers were forced to learn how to use Microsoft's software that came along with it. This created an inertia in Microsoft's favour. The next software these customers chose was more likely to be Microsoft's, not because their software was necessarily the best, but because most people were too busy to learn how to use anything else.

Microsoft's success and market share may differ from the rest by several orders of magnitude, but the difference was really enabled by Gate's early fortune, reinforced by a strong success-breeds-success dynamic. Of course, Gates's talent and effort played important roles in the extreme success of Microsoft. But that's not enough for creating such an outlier. Talent and effort are likely to be less important than circumstances in the sense that he could not have been so successful without the latter.

One might argue that many exceptional performers still gained their exceptional skill through hard work, exceptional motivation or "grit", so they do not deserve to receive lower reward and praise. Some have even suggested that there is a magic number for greatness, a ten-year or 10,000-hour rule. Many professionals and experts did acquire their exceptional skill through persistent, deliberate practices. In fact, Gates' 10,000 hours learning computer programming as a teenager has been highlighted as one of the reasons for his success.

But detailed analyses of the case studies of experts often suggest that certain situational factors beyond the control of these exceptional performers also play an important role. For example, three national champions in table tennis came from the same street in a small suburb of one town in England.

This wasn't a coincidence or because there was nothing else to do but practise ping pong. It turns out that a famous table tennis coach, Peter Charters, happened to retire in this particular suburb. Many kids who lived on the same street as the retired coach were attracted to this sport because of him and three of them, after following the "10,000-hour rule", performed exceptionally well, including winning the national championship.

Their talent and efforts were, of course, essential for realising their exceptional performances. But without their early luck (having a reliable, high-quality coach and supportive families), simply practicing 10,000 hours without adequate feedback wouldn't likely lead a randomly picked child to become a national champion.

We could also imagine a child with superior talent in table tennis suffering from early bad luck, such as not having a capable coach or being in a country where being an athlete was not considered to be a promising career. Then they might never have a chance to realise their potential. The implication is that the more exceptional a performance is, the fewer meaningful, applicable lessons we can actually learn from the "winner".

When it comes to moderate performance, it seems much more likely that our intuition about success is correct. Conventional wisdom, such as "the harder I work the luckier I get" or "chance favours the prepared mind", makes perfect sense when talking about someone moving from poor to good performance. Going from good to great, however, is a different story.

Being in the right place (succeeding in a context where early outcome has an enduring impact) at the right time (having early luck) can be so important that it overwhelms merits. With this in mind there's a good case that we shouldn't just reward or imitate life's winners and expect to have similar success. But there is a case that the winners should consider imitating the likes of Gates (who became a philanthropist) or Warren Buffett (who argues that richer Americans should pay higher taxes) who have chosen to use their wealth and success to do good things. The winners who appreciate their luck and do not take it all deserve more of our respect.

Q. Which of the following examples best represent an outlier, as described in the passage?

Detailed Solution for CAT Mock Test - 17 - Question 1
An outlier, as described in the passage is one 'whose performances may be examples set apart from the system that everyone else works within.' This would mean someone with extraordinary talent, one who excels in a position, where others might fail. The character in option A does show talent and skill, but is not that extraordinary, as it shows the normal good outcome of hard work. Moreover, once the boy cracked the examination, everything else was bound to follow. B is incorrect as this is not very extraordinary or surprising, as we are given factors that led to his poor performance; this implies that the student himself was not lacking in hard work or intellect. C is incorrect as it presents a surprising situation, but is not extraordinary enough on the part of the student, with respect to his talent or skill. D presents the outlier as we can infer that most college dropouts do not go on to have rich or successful careers.
CAT Mock Test - 17 - Question 2

Directions: Read the following passage carefully and answer the questions that follow.

Bill Gates is a lot luckier than you might realise. He may be a very talented man who worked his way up from geek to the top spot on the list of the world's richest people. But his extreme success perhaps tells us more about the importance of circumstances beyond his control than it does about how skill and perseverance are rewarded.

We often fall for the idea that the exceptional performers are the most skilled or talented. But this is flawed. Exceptional performances tend to occur in exceptional circumstances. Top performers are often the luckiest people, who have benefited from being at the right place and right time. They are what we call outliers, whose performances may be examples set apart from the system that everyone else works within.

Many treat Gates, and other highly successful people like him, as deserving of huge attention and reward, as people from whom we could learn a lot about how to succeed. But assuming life's "winners" got there from performance alone is likely to lead to disappointment. Even if you could imitate everything Gates did, you would not be able to replicate his initial good fortune.

For example, Gates's upper-class background and private education enabled him to gain extra programming experience when less than 0.01% of his generation then had access to computers. His mother's social connection with IBM's chairman enabled him to gain a contract from the then-leading PC company that was crucial for establishing his software empire.

This is important because most customers who used IBM computers were forced to learn how to use Microsoft's software that came along with it. This created an inertia in Microsoft's favour. The next software these customers chose was more likely to be Microsoft's, not because their software was necessarily the best, but because most people were too busy to learn how to use anything else.

Microsoft's success and market share may differ from the rest by several orders of magnitude, but the difference was really enabled by Gate's early fortune, reinforced by a strong success-breeds-success dynamic. Of course, Gates's talent and effort played important roles in the extreme success of Microsoft. But that's not enough for creating such an outlier. Talent and effort are likely to be less important than circumstances in the sense that he could not have been so successful without the latter.

One might argue that many exceptional performers still gained their exceptional skill through hard work, exceptional motivation or "grit", so they do not deserve to receive lower reward and praise. Some have even suggested that there is a magic number for greatness, a ten-year or 10,000-hour rule. Many professionals and experts did acquire their exceptional skill through persistent, deliberate practices. In fact, Gates' 10,000 hours learning computer programming as a teenager has been highlighted as one of the reasons for his success.

But detailed analyses of the case studies of experts often suggest that certain situational factors beyond the control of these exceptional performers also play an important role. For example, three national champions in table tennis came from the same street in a small suburb of one town in England.

This wasn't a coincidence or because there was nothing else to do but practise ping pong. It turns out that a famous table tennis coach, Peter Charters, happened to retire in this particular suburb. Many kids who lived on the same street as the retired coach were attracted to this sport because of him and three of them, after following the "10,000-hour rule", performed exceptionally well, including winning the national championship.

Their talent and efforts were, of course, essential for realising their exceptional performances. But without their early luck (having a reliable, high-quality coach and supportive families), simply practicing 10,000 hours without adequate feedback wouldn't likely lead a randomly picked child to become a national champion.

We could also imagine a child with superior talent in table tennis suffering from early bad luck, such as not having a capable coach or being in a country where being an athlete was not considered to be a promising career. Then they might never have a chance to realise their potential. The implication is that the more exceptional a performance is, the fewer meaningful, applicable lessons we can actually learn from the "winner".

When it comes to moderate performance, it seems much more likely that our intuition about success is correct. Conventional wisdom, such as "the harder I work the luckier I get" or "chance favours the prepared mind", makes perfect sense when talking about someone moving from poor to good performance. Going from good to great, however, is a different story.

Being in the right place (succeeding in a context where early outcome has an enduring impact) at the right time (having early luck) can be so important that it overwhelms merits. With this in mind there's a good case that we shouldn't just reward or imitate life's winners and expect to have similar success. But there is a case that the winners should consider imitating the likes of Gates (who became a philanthropist) or Warren Buffett (who argues that richer Americans should pay higher taxes) who have chosen to use their wealth and success to do good things. The winners who appreciate their luck and do not take it all deserve more of our respect.

Q. Which of the following questions cannot be answered on the basis of the information given in the passage?

Detailed Solution for CAT Mock Test - 17 - Question 2
B is incorrect as this question is answered throughout the passage: 'Being in the right place (succeeding in a context where early outcome has an enduring impact) at the right time (having early luck) can be so important that it overwhelms merits. With this in mind there's a good case that we shouldn't just reward or imitate life's winners and expect to have similar success.'

C is incorrect as this question is answered in the last few lines: 'The winners who appreciate their luck and do not take it all deserve more of our respect.'

D is incorrect as this question is answered in the first few paragraphs: ', Gates's upper-class background and private education enabled him to gain extra programming experience when less than 0.01% of his generation then had access to computers. His mother's social connection with IBM's chairman enabled him to gain a contract from the then-leading PC company that was crucial for establishing his software empire.'

A is the right answer, as the passage only tells us what outliers are, not the conditions that must be fulfilled in order to be characterised as one.

1 Crore+ students have signed up on EduRev. Have you? Download the App
CAT Mock Test - 17 - Question 3

Directions: Read the following passage carefully and answer the questions that follow.

Bill Gates is a lot luckier than you might realise. He may be a very talented man who worked his way up from geek to the top spot on the list of the world's richest people. But his extreme success perhaps tells us more about the importance of circumstances beyond his control than it does about how skill and perseverance are rewarded.

We often fall for the idea that the exceptional performers are the most skilled or talented. But this is flawed. Exceptional performances tend to occur in exceptional circumstances. Top performers are often the luckiest people, who have benefited from being at the right place and right time. They are what we call outliers, whose performances may be examples set apart from the system that everyone else works within.

Many treat Gates, and other highly successful people like him, as deserving of huge attention and reward, as people from whom we could learn a lot about how to succeed. But assuming life's "winners" got there from performance alone is likely to lead to disappointment. Even if you could imitate everything Gates did, you would not be able to replicate his initial good fortune.

For example, Gates's upper-class background and private education enabled him to gain extra programming experience when less than 0.01% of his generation then had access to computers. His mother's social connection with IBM's chairman enabled him to gain a contract from the then-leading PC company that was crucial for establishing his software empire.

This is important because most customers who used IBM computers were forced to learn how to use Microsoft's software that came along with it. This created an inertia in Microsoft's favour. The next software these customers chose was more likely to be Microsoft's, not because their software was necessarily the best, but because most people were too busy to learn how to use anything else.

Microsoft's success and market share may differ from the rest by several orders of magnitude, but the difference was really enabled by Gate's early fortune, reinforced by a strong success-breeds-success dynamic. Of course, Gates's talent and effort played important roles in the extreme success of Microsoft. But that's not enough for creating such an outlier. Talent and effort are likely to be less important than circumstances in the sense that he could not have been so successful without the latter.

One might argue that many exceptional performers still gained their exceptional skill through hard work, exceptional motivation or "grit", so they do not deserve to receive lower reward and praise. Some have even suggested that there is a magic number for greatness, a ten-year or 10,000-hour rule. Many professionals and experts did acquire their exceptional skill through persistent, deliberate practices. In fact, Gates' 10,000 hours learning computer programming as a teenager has been highlighted as one of the reasons for his success.

But detailed analyses of the case studies of experts often suggest that certain situational factors beyond the control of these exceptional performers also play an important role. For example, three national champions in table tennis came from the same street in a small suburb of one town in England.

This wasn't a coincidence or because there was nothing else to do but practise ping pong. It turns out that a famous table tennis coach, Peter Charters, happened to retire in this particular suburb. Many kids who lived on the same street as the retired coach were attracted to this sport because of him and three of them, after following the "10,000-hour rule", performed exceptionally well, including winning the national championship.

Their talent and efforts were, of course, essential for realising their exceptional performances. But without their early luck (having a reliable, high-quality coach and supportive families), simply practicing 10,000 hours without adequate feedback wouldn't likely lead a randomly picked child to become a national champion.

We could also imagine a child with superior talent in table tennis suffering from early bad luck, such as not having a capable coach or being in a country where being an athlete was not considered to be a promising career. Then they might never have a chance to realise their potential. The implication is that the more exceptional a performance is, the fewer meaningful, applicable lessons we can actually learn from the "winner".

When it comes to moderate performance, it seems much more likely that our intuition about success is correct. Conventional wisdom, such as "the harder I work the luckier I get" or "chance favours the prepared mind", makes perfect sense when talking about someone moving from poor to good performance. Going from good to great, however, is a different story.

Being in the right place (succeeding in a context where early outcome has an enduring impact) at the right time (having early luck) can be so important that it overwhelms merits. With this in mind there's a good case that we shouldn't just reward or imitate life's winners and expect to have similar success. But there is a case that the winners should consider imitating the likes of Gates (who became a philanthropist) or Warren Buffett (who argues that richer Americans should pay higher taxes) who have chosen to use their wealth and success to do good things. The winners who appreciate their luck and do not take it all deserve more of our respect.

Q. It can be understood that the main purpose of the author in the third paragraph is to:

Detailed Solution for CAT Mock Test - 17 - Question 3
A is incorrect as the author does not directly say or imply that attention and respect should not be given to winners. Rather he focuses more on the suggestion that one must not fall into the trap of imitating them and expecting similar returns. C is incorrect as the passage talks of all winners, not only Gates, although it does talk of him as an example. D is incorrect as this is his primary purpose in later paragraphs, not the third one. B is the right answer, as the author seeks to clear the misconception that success cannot be achieved simply by imitating life's winners, as they did not succeed through performance alone.
CAT Mock Test - 17 - Question 4

Directions: Read the following passage carefully and answer the questions that follow.

Bill Gates is a lot luckier than you might realise. He may be a very talented man who worked his way up from geek to the top spot on the list of the world's richest people. But his extreme success perhaps tells us more about the importance of circumstances beyond his control than it does about how skill and perseverance are rewarded.

We often fall for the idea that the exceptional performers are the most skilled or talented. But this is flawed. Exceptional performances tend to occur in exceptional circumstances. Top performers are often the luckiest people, who have benefited from being at the right place and right time. They are what we call outliers, whose performances may be examples set apart from the system that everyone else works within.

Many treat Gates, and other highly successful people like him, as deserving of huge attention and reward, as people from whom we could learn a lot about how to succeed. But assuming life's "winners" got there from performance alone is likely to lead to disappointment. Even if you could imitate everything Gates did, you would not be able to replicate his initial good fortune.

For example, Gates's upper-class background and private education enabled him to gain extra programming experience when less than 0.01% of his generation then had access to computers. His mother's social connection with IBM's chairman enabled him to gain a contract from the then-leading PC company that was crucial for establishing his software empire.

This is important because most customers who used IBM computers were forced to learn how to use Microsoft's software that came along with it. This created an inertia in Microsoft's favour. The next software these customers chose was more likely to be Microsoft's, not because their software was necessarily the best, but because most people were too busy to learn how to use anything else.

Microsoft's success and market share may differ from the rest by several orders of magnitude, but the difference was really enabled by Gate's early fortune, reinforced by a strong success-breeds-success dynamic. Of course, Gates's talent and effort played important roles in the extreme success of Microsoft. But that's not enough for creating such an outlier. Talent and effort are likely to be less important than circumstances in the sense that he could not have been so successful without the latter.

One might argue that many exceptional performers still gained their exceptional skill through hard work, exceptional motivation or "grit", so they do not deserve to receive lower reward and praise. Some have even suggested that there is a magic number for greatness, a ten-year or 10,000-hour rule. Many professionals and experts did acquire their exceptional skill through persistent, deliberate practices. In fact, Gates' 10,000 hours learning computer programming as a teenager has been highlighted as one of the reasons for his success.

But detailed analyses of the case studies of experts often suggest that certain situational factors beyond the control of these exceptional performers also play an important role. For example, three national champions in table tennis came from the same street in a small suburb of one town in England.

This wasn't a coincidence or because there was nothing else to do but practise ping pong. It turns out that a famous table tennis coach, Peter Charters, happened to retire in this particular suburb. Many kids who lived on the same street as the retired coach were attracted to this sport because of him and three of them, after following the "10,000-hour rule", performed exceptionally well, including winning the national championship.

Their talent and efforts were, of course, essential for realising their exceptional performances. But without their early luck (having a reliable, high-quality coach and supportive families), simply practicing 10,000 hours without adequate feedback wouldn't likely lead a randomly picked child to become a national champion.

We could also imagine a child with superior talent in table tennis suffering from early bad luck, such as not having a capable coach or being in a country where being an athlete was not considered to be a promising career. Then they might never have a chance to realise their potential. The implication is that the more exceptional a performance is, the fewer meaningful, applicable lessons we can actually learn from the "winner".

When it comes to moderate performance, it seems much more likely that our intuition about success is correct. Conventional wisdom, such as "the harder I work the luckier I get" or "chance favours the prepared mind", makes perfect sense when talking about someone moving from poor to good performance. Going from good to great, however, is a different story.

Being in the right place (succeeding in a context where early outcome has an enduring impact) at the right time (having early luck) can be so important that it overwhelms merits. With this in mind there's a good case that we shouldn't just reward or imitate life's winners and expect to have similar success. But there is a case that the winners should consider imitating the likes of Gates (who became a philanthropist) or Warren Buffett (who argues that richer Americans should pay higher taxes) who have chosen to use their wealth and success to do good things. The winners who appreciate their luck and do not take it all deserve more of our respect.

Q. Which of the following statements best sum up the author's view of Bill Gates and his success?

Detailed Solution for CAT Mock Test - 17 - Question 4
A is incorrect as the passage makes no mention of the fact the young people are misguided, he simply seeks to clear a misconception. B is incorrect as the author does acknowledge Gates' talent, and does not call him an opportunist. D is incorrect as it attributes all of his success to luck, and makes no mention of his own qualities. C is the right answer, as it best sums up the view the author has of Gates.
CAT Mock Test - 17 - Question 5

Directions: Read the following passage carefully and answer the questions that follow.

After many years of practising Buddhism in Thailand, my experience expands beyond the immediate community in Sri Racha. In recent years as I have visited the White Dragon Temple, the social unrest in Thailand has crept into the religious aspect of my trips.

Religion exists as an innate piece of the landscape that etches itself into the small details of Thailand. It occupies both a very physical presence within the community and also a mental one. According to the Office of National Buddhism, 40,717 Buddhist temples exist in Thailand. Of these temples, a large portion resides in Bangkok, Thailand's capital.

Aside from being an important tourist element, Buddhism plays an important part in the lives of Thai people - an estimated 94% of all Thai people practice Buddhism in the country according to a Central Intelligence Agency report. Time and time again, there have been movements - in 1997, 2007, and 2014 - to concretize Buddhism as the nation's official religion. The Thai Constitutional Drafting Committee (CDC) has, however, remained neutral in the relationship between the state and religion.

Though the government's ideological stance on religion is decidedly impartial, significant ripples exist in this seemingly placid surface, and religion morphs into a central focal point in many instances, whether the Thai government takes an intimate position on it or not. Faith remains a link to the personal lives of common citizens and royalty alike. King Bhumibol's funeral on October 14, 2016 featured traditional Buddhist funeral rites with the ritualistic bathing of the king's body and the chanting of orange-robed monks. Adding to this ceremonious burial, his body resided in the Temple of the Emerald Buddha so that people could pay their respects to the revered king, who provided stability for his country for 70 years. Though the king in Thailand did not hold any true, legislative power, he was a reverential symbol for the people of the country. His majesty's death occurred at a moment of tension in the country as a number of attacks rocked Thailand and has only caused this pressure to spill-over. Religion is something that connects people in Thailand yet, at the same time, can be a divisive element as is evident from attacks that have occurred in the nation over the last few years.

In the span of less than a day between August 11th and 12th of 2016, 11 bombings hit five provinces in Thailand, killing at least four Thai nationals and injuring 36. These bombings occurred almost a year after one of the most devastating attacks in Thai history in Bangkok, which killed 20 people and wounded 125 more. What's more, these attacks coincided with the Queen Sirikit's birthday. On August 17, 2015, Uighur militants splintered the Thai state as they bombed the Erawan Shrine. Though the motives for the attack were more aimed at the states' repatriation of Uighur refugees, the targeting of the temple was calculated: not only is the area around the shrine a densely populated area but also, it is frequented by many tourists. These acts of terrorism that assail the kingdom have left many Thais scared and unsure in a time, without a unifying leader. Known epithetically as the "land of smiles," Thailand has had little to smile about of late.

In light of this tumultuous time in the nation's history, religious institutions like the White Dragon Temple became integral in steadying the country's course. Through the diligent service that the temple provides for the community, it is a rallying point for many frightened Thais. See Knok, the central spiritual leader in the temple, and his followers have proved to be a "stabilizing element in the wake of the King's death," especially in Sri Racha, by continuing with their public works projects - providing educational help, burial services, food distribution, and a variety of other support structures. These actions from local community leaders have started to mend the fractures that occur on a national level.

I returned to Thailand in August of 2016, during the bombings in the southern provinces of the country. On one day during this visit, I bagged fruit and food for followers and local community members alike. The cadence of shifting palates of food and thump of vegetables into bags kept time with my human tempo. With each bag I loaded onto the palates, I could measure the burden on the community of Sri Racha lift slightly. In the glimmering eyes of the young men that I worked with, I could see the brightness of Thailand's future. Beneath me, I could feel the flexing and contracting of a nation, not torn by conflict but ready to rebuild and strive onward if only for a moment.

Q. Which of the following statement(s) is/ are confirmed from the facts provided in the passage?

I. The lack of a strong and unifying political leader in Thailand has made the country stagger further when it comes to dealing with the recent acts of terrorism that have struck the country.

II. The recent terrorist attacks in Thailand are an example of how religion can play a major role in bringing people of a common culture together in difficult times.

III. Efforts to make Buddhism as the official religion of Thailand have so far not panned out, as the government does not let religion interfere with state matters.

Detailed Solution for CAT Mock Test - 17 - Question 5
I is confirmed from the information given in the fifth paragraph, which tells us that the acts of terrorism have left the Thai people unsure and scared, especially since there is no unifying leader. III is confirmed from the information provided in the third paragraph which tells us that there have been movements to make Buddhism the official religion; however, the state has remained neutral in the relationship between the state and religion, from which we infer that this is the reason why those movements have not been successful. II cannot be confirmed from the passage; on the contrary, it is contradicted by the information given in the fourth paragraph which tells us that the recent terrorism acts show how religion can play a divisive role. D is the right answer, as only I and III are correct.
CAT Mock Test - 17 - Question 6

Directions: Read the following passage carefully and answer the questions that follow.

After many years of practising Buddhism in Thailand, my experience expands beyond the immediate community in Sri Racha. In recent years as I have visited the White Dragon Temple, the social unrest in Thailand has crept into the religious aspect of my trips.

Religion exists as an innate piece of the landscape that etches itself into the small details of Thailand. It occupies both a very physical presence within the community and also a mental one. According to the Office of National Buddhism, 40,717 Buddhist temples exist in Thailand. Of these temples, a large portion resides in Bangkok, Thailand's capital.

Aside from being an important tourist element, Buddhism plays an important part in the lives of Thai people - an estimated 94% of all Thai people practice Buddhism in the country according to a Central Intelligence Agency report. Time and time again, there have been movements - in 1997, 2007, and 2014 - to concretize Buddhism as the nation's official religion. The Thai Constitutional Drafting Committee (CDC) has, however, remained neutral in the relationship between the state and religion.

Though the government's ideological stance on religion is decidedly impartial, significant ripples exist in this seemingly placid surface, and religion morphs into a central focal point in many instances, whether the Thai government takes an intimate position on it or not. Faith remains a link to the personal lives of common citizens and royalty alike. King Bhumibol's funeral on October 14, 2016 featured traditional Buddhist funeral rites with the ritualistic bathing of the king's body and the chanting of orange-robed monks. Adding to this ceremonious burial, his body resided in the Temple of the Emerald Buddha so that people could pay their respects to the revered king, who provided stability for his country for 70 years. Though the king in Thailand did not hold any true, legislative power, he was a reverential symbol for the people of the country. His majesty's death occurred at a moment of tension in the country as a number of attacks rocked Thailand and has only caused this pressure to spill-over. Religion is something that connects people in Thailand yet, at the same time, can be a divisive element as is evident from attacks that have occurred in the nation over the last few years.

In the span of less than a day between August 11th and 12th of 2016, 11 bombings hit five provinces in Thailand, killing at least four Thai nationals and injuring 36. These bombings occurred almost a year after one of the most devastating attacks in Thai history in Bangkok, which killed 20 people and wounded 125 more. What's more, these attacks coincided with the Queen Sirikit's birthday. On August 17, 2015, Uighur militants splintered the Thai state as they bombed the Erawan Shrine. Though the motives for the attack were more aimed at the states' repatriation of Uighur refugees, the targeting of the temple was calculated: not only is the area around the shrine a densely populated area but also, it is frequented by many tourists. These acts of terrorism that assail the kingdom have left many Thais scared and unsure in a time, without a unifying leader. Known epithetically as the "land of smiles," Thailand has had little to smile about of late.

In light of this tumultuous time in the nation's history, religious institutions like the White Dragon Temple became integral in steadying the country's course. Through the diligent service that the temple provides for the community, it is a rallying point for many frightened Thais. See Knok, the central spiritual leader in the temple, and his followers have proved to be a "stabilizing element in the wake of the King's death," especially in Sri Racha, by continuing with their public works projects - providing educational help, burial services, food distribution, and a variety of other support structures. These actions from local community leaders have started to mend the fractures that occur on a national level.

I returned to Thailand in August of 2016, during the bombings in the southern provinces of the country. On one day during this visit, I bagged fruit and food for followers and local community members alike. The cadence of shifting palates of food and thump of vegetables into bags kept time with my human tempo. With each bag I loaded onto the palates, I could measure the burden on the community of Sri Racha lift slightly. In the glimmering eyes of the young men that I worked with, I could see the brightness of Thailand's future. Beneath me, I could feel the flexing and contracting of a nation, not torn by conflict but ready to rebuild and strive onward if only for a moment.

Q. Which of the following questions cannot be answered on the basis of the information given in the passage?

Detailed Solution for CAT Mock Test - 17 - Question 6
A can be answered from the information given in the second paragraph, which tells us about the presence of numerous Buddhist temples, which would show a physical presence of religion in the community. B can be answered from the info given in the fourth paragraph, which tells us that the king's death occurred at a moment of tension. The paragraph then goes into the details of this moment of tension. D can be answered from the information given in the fifth paragraph, which tells us that the attack was "more aimed at the states' repatriation of Uighur refugees". C cannot be answered from the passage, as the only mention of religion and tourism is made in the third paragraph, which simply states that Buddhism is an important tourist element, but does not venture into the details of how it is so. C is the right answer.
CAT Mock Test - 17 - Question 7

Directions: Read the following passage carefully and answer the questions that follow.

After many years of practising Buddhism in Thailand, my experience expands beyond the immediate community in Sri Racha. In recent years as I have visited the White Dragon Temple, the social unrest in Thailand has crept into the religious aspect of my trips.

Religion exists as an innate piece of the landscape that etches itself into the small details of Thailand. It occupies both a very physical presence within the community and also a mental one. According to the Office of National Buddhism, 40,717 Buddhist temples exist in Thailand. Of these temples, a large portion resides in Bangkok, Thailand's capital.

Aside from being an important tourist element, Buddhism plays an important part in the lives of Thai people - an estimated 94% of all Thai people practice Buddhism in the country according to a Central Intelligence Agency report. Time and time again, there have been movements - in 1997, 2007, and 2014 - to concretize Buddhism as the nation's official religion. The Thai Constitutional Drafting Committee (CDC) has, however, remained neutral in the relationship between the state and religion.

Though the government's ideological stance on religion is decidedly impartial, significant ripples exist in this seemingly placid surface, and religion morphs into a central focal point in many instances, whether the Thai government takes an intimate position on it or not. Faith remains a link to the personal lives of common citizens and royalty alike. King Bhumibol's funeral on October 14, 2016 featured traditional Buddhist funeral rites with the ritualistic bathing of the king's body and the chanting of orange-robed monks. Adding to this ceremonious burial, his body resided in the Temple of the Emerald Buddha so that people could pay their respects to the revered king, who provided stability for his country for 70 years. Though the king in Thailand did not hold any true, legislative power, he was a reverential symbol for the people of the country. His majesty's death occurred at a moment of tension in the country as a number of attacks rocked Thailand and has only caused this pressure to spill-over. Religion is something that connects people in Thailand yet, at the same time, can be a divisive element as is evident from attacks that have occurred in the nation over the last few years.

In the span of less than a day between August 11th and 12th of 2016, 11 bombings hit five provinces in Thailand, killing at least four Thai nationals and injuring 36. These bombings occurred almost a year after one of the most devastating attacks in Thai history in Bangkok, which killed 20 people and wounded 125 more. What's more, these attacks coincided with the Queen Sirikit's birthday. On August 17, 2015, Uighur militants splintered the Thai state as they bombed the Erawan Shrine. Though the motives for the attack were more aimed at the states' repatriation of Uighur refugees, the targeting of the temple was calculated: not only is the area around the shrine a densely populated area but also, it is frequented by many tourists. These acts of terrorism that assail the kingdom have left many Thais scared and unsure in a time, without a unifying leader. Known epithetically as the "land of smiles," Thailand has had little to smile about of late.

In light of this tumultuous time in the nation's history, religious institutions like the White Dragon Temple became integral in steadying the country's course. Through the diligent service that the temple provides for the community, it is a rallying point for many frightened Thais. See Knok, the central spiritual leader in the temple, and his followers have proved to be a "stabilizing element in the wake of the King's death," especially in Sri Racha, by continuing with their public works projects - providing educational help, burial services, food distribution, and a variety of other support structures. These actions from local community leaders have started to mend the fractures that occur on a national level.

I returned to Thailand in August of 2016, during the bombings in the southern provinces of the country. On one day during this visit, I bagged fruit and food for followers and local community members alike. The cadence of shifting palates of food and thump of vegetables into bags kept time with my human tempo. With each bag I loaded onto the palates, I could measure the burden on the community of Sri Racha lift slightly. In the glimmering eyes of the young men that I worked with, I could see the brightness of Thailand's future. Beneath me, I could feel the flexing and contracting of a nation, not torn by conflict but ready to rebuild and strive onward if only for a moment.

Q. Which of the following can be said to be the central idea of the sixth paragraph?

Detailed Solution for CAT Mock Test - 17 - Question 7
A is incorrect as the paragraph does not talk of the government's role. B is incorrect as the paragraph does not talk about the role played by local people, rather we can say that it talks about the role of local community leaders. D is incorrect as it is too specific and restricts itself to the White Dragon Temple, while the paragraph only seeks to illustrate the role played by religious institutions by means of giving an example of one such institution. C is the right answer.
CAT Mock Test - 17 - Question 8

Directions: Read the following passage carefully and answer the questions that follow.

After many years of practising Buddhism in Thailand, my experience expands beyond the immediate community in Sri Racha. In recent years as I have visited the White Dragon Temple, the social unrest in Thailand has crept into the religious aspect of my trips.

Religion exists as an innate piece of the landscape that etches itself into the small details of Thailand. It occupies both a very physical presence within the community and also a mental one. According to the Office of National Buddhism, 40,717 Buddhist temples exist in Thailand. Of these temples, a large portion resides in Bangkok, Thailand's capital.

Aside from being an important tourist element, Buddhism plays an important part in the lives of Thai people - an estimated 94% of all Thai people practice Buddhism in the country according to a Central Intelligence Agency report. Time and time again, there have been movements - in 1997, 2007, and 2014 - to concretize Buddhism as the nation's official religion. The Thai Constitutional Drafting Committee (CDC) has, however, remained neutral in the relationship between the state and religion.

Though the government's ideological stance on religion is decidedly impartial, significant ripples exist in this seemingly placid surface, and religion morphs into a central focal point in many instances, whether the Thai government takes an intimate position on it or not. Faith remains a link to the personal lives of common citizens and royalty alike. King Bhumibol's funeral on October 14, 2016 featured traditional Buddhist funeral rites with the ritualistic bathing of the king's body and the chanting of orange-robed monks. Adding to this ceremonious burial, his body resided in the Temple of the Emerald Buddha so that people could pay their respects to the revered king, who provided stability for his country for 70 years. Though the king in Thailand did not hold any true, legislative power, he was a reverential symbol for the people of the country. His majesty's death occurred at a moment of tension in the country as a number of attacks rocked Thailand and has only caused this pressure to spill-over. Religion is something that connects people in Thailand yet, at the same time, can be a divisive element as is evident from attacks that have occurred in the nation over the last few years.

In the span of less than a day between August 11th and 12th of 2016, 11 bombings hit five provinces in Thailand, killing at least four Thai nationals and injuring 36. These bombings occurred almost a year after one of the most devastating attacks in Thai history in Bangkok, which killed 20 people and wounded 125 more. What's more, these attacks coincided with the Queen Sirikit's birthday. On August 17, 2015, Uighur militants splintered the Thai state as they bombed the Erawan Shrine. Though the motives for the attack were more aimed at the states' repatriation of Uighur refugees, the targeting of the temple was calculated: not only is the area around the shrine a densely populated area but also, it is frequented by many tourists. These acts of terrorism that assail the kingdom have left many Thais scared and unsure in a time, without a unifying leader. Known epithetically as the "land of smiles," Thailand has had little to smile about of late.

In light of this tumultuous time in the nation's history, religious institutions like the White Dragon Temple became integral in steadying the country's course. Through the diligent service that the temple provides for the community, it is a rallying point for many frightened Thais. See Knok, the central spiritual leader in the temple, and his followers have proved to be a "stabilizing element in the wake of the King's death," especially in Sri Racha, by continuing with their public works projects - providing educational help, burial services, food distribution, and a variety of other support structures. These actions from local community leaders have started to mend the fractures that occur on a national level.

I returned to Thailand in August of 2016, during the bombings in the southern provinces of the country. On one day during this visit, I bagged fruit and food for followers and local community members alike. The cadence of shifting palates of food and thump of vegetables into bags kept time with my human tempo. With each bag I loaded onto the palates, I could measure the burden on the community of Sri Racha lift slightly. In the glimmering eyes of the young men that I worked with, I could see the brightness of Thailand's future. Beneath me, I could feel the flexing and contracting of a nation, not torn by conflict but ready to rebuild and strive onward if only for a moment.

Q. What is the main purpose behind the author's words when he says: "Known epithetically as the "land of smiles," Thailand has had little to smile about of late"?

Detailed Solution for CAT Mock Test - 17 - Question 8
A is incorrect as the author does not talk of hypocrisy of the Thai people; he does not in any way hold them responsible for not living up to their name. Instead, by means od examples in the next paragraph, he seeks to delicately suggest that it is ironical and contrary to expectation that a country known for its smiles is facing circumstances that would disable its people to remain happy or smile. B is incorrect as although it is partially correct - the author does claim that the epithet no longer holds true for Thailand, he does not give us the origins of this epithet. C is the right answer, as it best sums up the main purpose that the author has in mind while using these words.
CAT Mock Test - 17 - Question 9

Directions: Read the following passage carefully and answer the questions that follow.

No language has spread as widely as English, and it continues to spread. Internationally the desire to learn it is insatiable. In the twenty-first century the world is becoming more urban and more middle class, and the adoption of English is a symptom of this, for increasingly English serves as the lingua franca of business and popular culture. It is dominant or at least very prominent in other areas such as shipping, diplomacy, computing, medicine and education. A recent study has suggested that among students in the United Arab Emirates "Arabic is associated with tradition, home, religion, culture, school, arts and social sciences," whereas English "is symbolic of modernity, respect, work, higher education, commerce, economics and science and technology."

Wherever English has been used, it has lasted. Cultural might outlives military rule. In the colonial period, the languages of settlers dominated the languages of the peoples whose land they seized. They marginalized them and in some cases eventually drove them to extinction. All the while they absorbed from them whatever local terms seemed useful. The colonists' languages practised a sort of cannibalism, and its legacy is still sharply felt. English is treated with suspicion in many places where it was once the language of the imperial overlords. It is far from being a force for unity, and its endurance is stressful. In India, while English is much used in the media, administration, education and business, there are calls to curb its influence. Yet even where English has been denigrated as an instrument of colonialism, it has held on - and in most cases grown, increasing its numbers of speakers and functions.

Today it is English, rather than any created alternative, that is the world's auxiliary tongue. There are more people who use English as a second language than there are native speakers. Estimates of the numbers vary, but even the most guarded view is that English has 500 million second-language speakers. Far more of the world's citizens are eagerly jumping on board than trying to resist its progress. In places where English is used as a second language, its users often perceive it as free from the limitations of their native languages. They associate it with power and social status, and see it as a supple and sensuous medium for self-expression. It symbolizes choice and liberty. But while many of those who do not have a grasp of the language aspire to learn it, there are many others who perceive it as an instrument of oppression, associated not only with imperialism but also with the predations of capitalism and Christianity.

There are challenges to the position of English as the dominant world language in the twenty-first century. The main ones seem likely to come from Spanish and Mandarin Chinese. Both have more first-language users than English. But at present neither is much used as a lingua franca. The majority of speakers of Mandarin Chinese live in one country, and, excepting Spain, most Spanish-speakers are in the Americas. Two challenges stand out. I have mentioned India already; English is important to its global ambitions. The language's roots there are colonial, but English connects Indians less to the past than to the future. Already the language is used by more people in India than in any other country, the United States included. Meanwhile in China the number of students learning the language is increasing rapidly. The entrepreneur Li Yang has developed Crazy English, an unorthodox teaching method. It involves a lot of shouting. This, Li explains, is the way for Chinese to activate their "international muscles." His agenda is patriotic.

The embrace of English in the world's two most populous countries means that the language is changing. Some of the changes are likely to prove disconcerting for its native speakers. The "English-ness" of English is being diluted. So, more surprisingly, is its American flavour. English's centre of gravity is moving; in fact, in the twenty-first century the language has many centres. As this continues, native English-speakers may find themselves at a disadvantage.

At the same time, native speakers of English tend to assume that their ability in this potent language makes it unimportant to learn other languages. The reality is different. British companies often miss out on export opportunities because of a lack of relevant language skills. Moreover, there is a chance that a command of English will within twenty or thirty years be regarded as a basic skill for business, and native speakers of the language will no longer enjoy any competitive advantage. The consequences are complex. Some, it would seem, are not as intended. Even as vast amounts are spent on spreading British English, the reality is that English is taking on more and more local colour in the different places where it is used. Accordingly, while the number of languages in the world is diminishing, the number of Englishes is increasing.

Q. It can be understood from the context of the passage that when the author says: "the number of Englishes is increasing", he means which of the following?

Detailed Solution for CAT Mock Test - 17 - Question 9
Note that the sentence prior to this one talks about English taking on more and more local colour in the different places where it is used. The context of the last and the second to last paragraph also talks along these lines where it illustrates how English is changing as more and more people are speaking it, and how local forms are not dominating the realm of spoken English. From this we can ascertain that B is most likely to be the meaning behind the author's words. B is the right answer.
CAT Mock Test - 17 - Question 10

No language has spread as widely as English, and it continues to spread. Internationally the desire to learn it is insatiable. In the twenty-first century the world is becoming more urban and more middle class, and the adoption of English is a symptom of this, for increasingly English serves as the lingua franca of business and popular culture. It is dominant or at least very prominent in other areas such as shipping, diplomacy, computing, medicine and education. A recent study has suggested that among students in the United Arab Emirates "Arabic is associated with tradition, home, religion, culture, school, arts and social sciences," whereas English "is symbolic of modernity, respect, work, higher education, commerce, economics and science and technology."

Wherever English has been used, it has lasted. Cultural might outlives military rule. In the colonial period, the languages of settlers dominated the languages of the peoples whose land they seized. They marginalized them and in some cases eventually drove them to extinction. All the while they absorbed from them whatever local terms seemed useful. The colonists' languages practised a sort of cannibalism, and its legacy is still sharply felt. English is treated with suspicion in many places where it was once the language of the imperial overlords. It is far from being a force for unity, and its endurance is stressful. In India, while English is much used in the media, administration, education and business, there are calls to curb its influence. Yet even where English has been denigrated as an instrument of colonialism, it has held on - and in most cases grown, increasing its numbers of speakers and functions.

Today it is English, rather than any created alternative, that is the world's auxiliary tongue. There are more people who use English as a second language than there are native speakers. Estimates of the numbers vary, but even the most guarded view is that English has 500 million second-language speakers. Far more of the world's citizens are eagerly jumping on board than trying to resist its progress. In places where English is used as a second language, its users often perceive it as free from the limitations of their native languages. They associate it with power and social status, and see it as a supple and sensuous medium for self-expression. It symbolizes choice and liberty. But while many of those who do not have a grasp of the language aspire to learn it, there are many others who perceive it as an instrument of oppression, associated not only with imperialism but also with the predators of capitalism and Christianity.

There are challenges to the position of English as the dominant world language in the twenty-first century. The main ones seem likely to come from Spanish and Mandarin Chinese. Both have more first-language users than English. But at present neither is much used as a lingua franca. The majority of speakers of Mandarin Chinese live in one country, and, excepting Spain, most Spanish-speakers are in the Americas. Two challenges stand out. I have mentioned India already; English is important to its global ambitions. The language's roots there are colonial, but English connects Indians less to the past than to the future. Already the language is used by more people in India than in any other country, the United States included. Meanwhile in China the number of students learning the language is increasing rapidly. The entrepreneur Li Yang has developed Crazy English, an unorthodox teaching method. It involves a lot of shouting. This, Li explains, is the way for Chinese to activate their "international muscles." His agenda is patriotic.

The embrace of English in the world's two most populous countries means that the language is changing. Some of the changes are likely to prove disconcerting for its native speakers. The "English-ness" of English is being diluted. So, more surprisingly, is its American flavour. English's centre of gravity is moving; in fact, in the twenty-first century the language has many centres. As this continues, native English-speakers may find themselves at a disadvantage.

At the same time, native speakers of English tend to assume that their ability in this potent language makes it unimportant to learn other languages. The reality is different. British companies often miss out on export opportunities because of a lack of relevant language skills. Moreover, there is a chance that a command of English will within twenty or thirty years be regarded as a basic skill for business, and native speakers of the language will no longer enjoy any competitive advantage. The consequences are complex. Some, it would seem, are not as intended. Even as vast amounts are spent on spreading British English, the reality is that English is taking on more and more local colour in the different places where it is used. Accordingly, while the number of languages in the world is diminishing, the number of Englishes is increasing.

Q. The passage is most likely to be an excerpt taken from which of the following?

Detailed Solution for CAT Mock Test - 17 - Question 10
B is unlikely to be the answer, as the passage lacks the use of personal pronouns (I, you, me, we, us, etc.), conversational language and it does not address the listeners. C is incorrect as the passage does not involve the author's own research, studies or experiments. It only cites some studies. D is incorrect as the passage does not discuss events or happenings. A is the most likely answer, as the passage revolves around a single topic, and uses examples, studies and facts to illustrate it.
CAT Mock Test - 17 - Question 11

No language has spread as widely as English, and it continues to spread. Internationally the desire to learn it is insatiable. In the twenty-first century the world is becoming more urban and more middle class, and the adoption of English is a symptom of this, for increasingly English serves as the lingua franca of business and popular culture. It is dominant or at least very prominent in other areas such as shipping, diplomacy, computing, medicine and education. A recent study has suggested that among students in the United Arab Emirates "Arabic is associated with tradition, home, religion, culture, school, arts and social sciences," whereas English "is symbolic of modernity, respect, work, higher education, commerce, economics and science and technology."

Wherever English has been used, it has lasted. Cultural might outlives military rule. In the colonial period, the languages of settlers dominated the languages of the peoples whose land they seized. They marginalized them and in some cases eventually drove them to extinction. All the while they absorbed from them whatever local terms seemed useful. The colonists' languages practised a sort of cannibalism, and its legacy is still sharply felt. English is treated with suspicion in many places where it was once the language of the imperial overlords. It is far from being a force for unity, and its endurance is stressful. In India, while English is much used in the media, administration, education and business, there are calls to curb its influence. Yet even where English has been denigrated as an instrument of colonialism, it has held on - and in most cases grown, increasing its numbers of speakers and functions.

Today it is English, rather than any created alternative, that is the world's auxiliary tongue. There are more people who use English as a second language than there are native speakers. Estimates of the numbers vary, but even the most guarded view is that English has 500 million second-language speakers. Far more of the world's citizens are eagerly jumping on board than trying to resist its progress. In places where English is used as a second language, its users often perceive it as free from the limitations of their native languages. They associate it with power and social status, and see it as a supple and sensuous medium for self-expression. It symbolizes choice and liberty. But while many of those who do not have a grasp of the language aspire to learn it, there are many others who perceive it as an instrument of oppression, associated not only with imperialism but also with the predations of capitalism and Christianity.

There are challenges to the position of English as the dominant world language in the twenty-first century. The main ones seem likely to come from Spanish and Mandarin Chinese. Both have more first-language users than English. But at present neither is much used as a lingua franca. The majority of speakers of Mandarin Chinese live in one country, and, excepting Spain, most Spanish-speakers are in the Americas. Two challenges stand out. I have mentioned India already; English is important to its global ambitions. The language's roots there are colonial, but English connects Indians less to the past than to the future. Already the language is used by more people in India than in any other country, the United States included. Meanwhile in China the number of students learning the language is increasing rapidly. The entrepreneur Li Yang has developed Crazy English, an unorthodox teaching method. It involves a lot of shouting. This, Li explains, is the way for Chinese to activate their "international muscles." His agenda is patriotic.

The embrace of English in the world's two most populous countries means that the language is changing. Some of the changes are likely to prove disconcerting for its native speakers. The "English-ness" of English is being diluted. So, more surprisingly, is its American flavour. English's centre of gravity is moving; in fact, in the twenty-first century the language has many centres. As this continues, native English-speakers may find themselves at a disadvantage.

At the same time, native speakers of English tend to assume that their ability in this potent language makes it unimportant to learn other languages. The reality is different. British companies often miss out on export opportunities because of a lack of relevant language skills. Moreover, there is a chance that a command of English will within twenty or thirty years be regarded as a basic skill for business, and native speakers of the language will no longer enjoy any competitive advantage. The consequences are complex. Some, it would seem, are not as intended. Even as vast amounts are spent on spreading British English, the reality is that English is taking on more and more local colour in the different places where it is used. Accordingly, while the number of languages in the world is diminishing, the number of Englishes is increasing.

Q. Which of the following would be the most suitable title for the above passage?

Detailed Solution for CAT Mock Test - 17 - Question 11
A is incorrect as the passage does not highlight the importance of learning English, nor does it seek to advise the user to do so; it only talks about the importance the language is going to hold in coming years. B is incorrect as this is too specific and forms only one part of the passage, and is nowhere near the main idea around which the passage revolves. C is incorrect as the passage only talks briefly about the speakers of the language as an accessory to the main topic of discussion. D is the right answer, as it best sums up the central idea, and provides a suitable title for the passage.
CAT Mock Test - 17 - Question 12

No language has spread as widely as English, and it continues to spread. Internationally the desire to learn it is insatiable. In the twenty-first century the world is becoming more urban and more middle class, and the adoption of English is a symptom of this, for increasingly English serves as the lingua franca of business and popular culture. It is dominant or at least very prominent in other areas such as shipping, diplomacy, computing, medicine and education. A recent study has suggested that among students in the United Arab Emirates "Arabic is associated with tradition, home, religion, culture, school, arts and social sciences," whereas English "is symbolic of modernity, respect, work, higher education, commerce, economics and science and technology."

Wherever English has been used, it has lasted. Cultural might outlives military rule. In the colonial period, the languages of settlers dominated the languages of the peoples whose land they seized. They marginalized them and in some cases eventually drove them to extinction. All the while they absorbed from them whatever local terms seemed useful. The colonists' languages practised a sort of cannibalism, and its legacy is still sharply felt. English is treated with suspicion in many places where it was once the language of the imperial overlords. It is far from being a force for unity, and its endurance is stressful. In India, while English is much used in the media, administration, education and business, there are calls to curb its influence. Yet even where English has been denigrated as an instrument of colonialism, it has held on - and in most cases grown, increasing its numbers of speakers and functions.

Today it is English, rather than any created alternative, that is the world's auxiliary tongue. There are more people who use English as a second language than there are native speakers. Estimates of the numbers vary, but even the most guarded view is that English has 500 million second-language speakers. Far more of the world's citizens are eagerly jumping on board than trying to resist its progress. In places where English is used as a second language, its users often perceive it as free from the limitations of their native languages. They associate it with power and social status, and see it as a supple and sensuous medium for self-expression. It symbolizes choice and liberty. But while many of those who do not have a grasp of the language aspire to learn it, there are many others who perceive it as an instrument of oppression, associated not only with imperialism but also with the predations of capitalism and Christianity.

There are challenges to the position of English as the dominant world language in the twenty-first century. The main ones seem likely to come from Spanish and Mandarin Chinese. Both have more first-language users than English. But at present neither is much used as a lingua franca. The majority of speakers of Mandarin Chinese live in one country, and, excepting Spain, most Spanish-speakers are in the Americas. Two challenges stand out. I have mentioned India already; English is important to its global ambitions. The language's roots there are colonial, but English connects Indians less to the past than to the future. Already the language is used by more people in India than in any other country, the United States included. Meanwhile in China the number of students learning the language is increasing rapidly. The entrepreneur Li Yang has developed Crazy English, an unorthodox teaching method. It involves a lot of shouting. This, Li explains, is the way for Chinese to activate their "international muscles." His agenda is patriotic.

The embrace of English in the world's two most populous countries means that the language is changing. Some of the changes are likely to prove disconcerting for its native speakers. The "English-ness" of English is being diluted. So, more surprisingly, is its American flavour. English's centre of gravity is moving; in fact, in the twenty-first century the language has many centres. As this continues, native English-speakers may find themselves at a disadvantage.

At the same time, native speakers of English tend to assume that their ability in this potent language makes it unimportant to learn other languages. The reality is different. British companies often miss out on export opportunities because of a lack of relevant language skills. Moreover, there is a chance that a command of English will within twenty or thirty years be regarded as a basic skill for business, and native speakers of the language will no longer enjoy any competitive advantage. The consequences are complex. Some, it would seem, are not as intended. Even as vast amounts are spent on spreading British English, the reality is that English is taking on more and more local colour in the different places where it is used. Accordingly, while the number of languages in the world is diminishing, the number of Englishes is increasing.

Q. Which of the following questions cannot be answered on the basis of the information given in the passage?

Detailed Solution for CAT Mock Test - 17 - Question 12
A is answered by the information given in the fifth paragraph, which tells us how English will change, and how its 'English-ness' will be diluted. B is answered by the information given in the fourth paragraph, which tells us that users of English in countries where it is a second language associate it with power and social status. It goes into further details on the same lines. C is answered by the information provided in the third paragraph, which tells us how the language strengthened and expanded its hold in colonial times. D is the right answer, as the passage does not provide sufficient information to answer this question.
CAT Mock Test - 17 - Question 13

Passage: In all battles two things are usually required of the Commander—in—Chief: to make a good plan for his army and to keep a strong reserve. Both of these are also obligatory for the painter. To make a plan, thorough reconnaissance of the country where the battle is to be fought is needed. Its fields, its mountains, its rivers, its bridges, its trees, its flowers, its atmosphere—all require and repay attentive observation from a special point of view. I think this is one of the chief delights that have come to me through painting. No doubt many people who are lovers of art have acquired it to a high degree without actually practicing.
But I expect that nothing will make one observe more quickly or more thoroughly than having to face the difficulty of representing the thing observed. And mind you, if you do observe accurately and with refinement, and if you do record what you have seen with tolerable correspondence, the result follows on the canvas with startling obedience. But in order to make his plan, the General must not only reconnoiter the battle—ground; he must also study the achievements of the great Captains of the past. He must bring the observations he has collected in the field into comparison with the treatment of similar incidents by famous chiefs. Considering that, the galleries of Europe take on a new—and to me at least a severely practical—interest. "This, then, is how —— painted a cataract.
Exactly, and there is that same light I noticed last week in the waterfall at ——. " And so on. You see the difficulty that baffled you yesterday; and you see how easily it has been overcome by a great or even by a skillful painter. Not only is your observation of Nature sensibly improved and developed, but also your comprehension of the masterpieces of art. But it is in the use and withholding of their reserves that the great commanders have generally excelled. After all, when once the last reserve has been thrown in, the commander's part is played.
If that does not win the battle, he has nothing else to give. Everything must be left to luck and to the fighting troops. But these last reserves, in the absence of high direction, are apt to get into sad confusion, all mixed together in a nasty mess, without order or plan—and consequently without effect. Mere masses count no more. The largest brush, the brightest colors cannot even make an impression. The pictorial battlefield becomes a sea of mud mercifully veiled by the fog of war. Even though the General plunges in himself and emerges bespattered, as he sometimes does, he will not retrieve the day. In painting, the reserves consist in Proportion or Relation.
And it is here that the art of the painter marches along the road which is traversed by all the greatest harmonies in thought. At one side of the palette there is white, at the other black; and neither is ever used ?neat.' Between these two rigid limits all the action must lie, all the power required must be generated. Black and white themselves placed in juxtaposition make no great impression; and yet they are the most that you can do in pure contrast.

Q. The existence of which of the following would most strongly challenge the author's conception of the process of painting?

Detailed Solution for CAT Mock Test - 17 - Question 13

What does the author's "conception of the process of painting " consist of? Planning and backup. Search for an answer choice that violates one of those principles. Choice (C) describes a painting of something that can't be seen and therefore can't be planned for in the way the author describes. It's a winner.

Wrong answers:

(A): Opposite. Painting waves on the beach can be planned just like the waterfall that the author describes.

(B): Faulty Use of Detail. While the author mentions the advantages in planning painting gained by visiting European galleries, there's no reason why one would have to visit a European gallery especially to be great.

(D): Out of Scope. The answer choice deals with battle, but not any part of battle that the author compares to painting.

CAT Mock Test - 17 - Question 14

Passage: In all battles two things are usually required of the Commander—in—Chief: to make a good plan for his army and to keep a strong reserve. Both of these are also obligatory for the painter. To make a plan, thorough reconnaissance of the country where the battle is to be fought is needed. Its fields, its mountains, its rivers, its bridges, its trees, its flowers, its atmosphere—all require and repay attentive observation from a special point of view. I think this is one of the chief delights that have come to me through painting. No doubt many people who are lovers of art have acquired it to a high degree without actually practicing.
But I expect that nothing will make one observe more quickly or more thoroughly than having to face the difficulty of representing the thing observed. And mind you, if you do observe accurately and with refinement, and if you do record what you have seen with tolerable correspondence, the result follows on the canvas with startling obedience. But in order to make his plan, the General must not only reconnoiter the battle—ground; he must also study the achievements of the great Captains of the past. He must bring the observations he has collected in the field into comparison with the treatment of similar incidents by famous chiefs. Considering that, the galleries of Europe take on a new—and to me at least a severely practical—interest. "This, then, is how —— painted a cataract.
Exactly, and there is that same light I noticed last week in the waterfall at ——. " And so on. You see the difficulty that baffled you yesterday; and you see how easily it has been overcome by a great or even by a skillful painter. Not only is your observation of Nature sensibly improved and developed, but also your comprehension of the masterpieces of art. But it is in the use and withholding of their reserves that the great commanders have generally excelled. After all, when once the last reserve has been thrown in, the commander's part is played.
If that does not win the battle, he has nothing else to give. Everything must be left to luck and to the fighting troops. But these last reserves, in the absence of high direction, are apt to get into sad confusion, all mixed together in a nasty mess, without order or plan—and consequently without effect. Mere masses count no more. The largest brush, the brightest colors cannot even make an impression. The pictorial battlefield becomes a sea of mud mercifully veiled by the fog of war. Even though the General plunges in himself and emerges bespattered, as he sometimes does, he will not retrieve the day. In painting, the reserves consist in Proportion or Relation.
And it is here that the art of the painter marches along the road which is traversed by all the greatest harmonies in thought. At one side of the palette there is white, at the other black; and neither is ever used ?neat.' Between these two rigid limits all the action must lie, all the power required must be generated. Black and white themselves placed in juxtaposition make no great impression; and yet they are the most that you can do in pure contrast.

Q. The passage suggests that having the finest art supplies at hand may NOT always be helpful to a painter because:

Detailed Solution for CAT Mock Test - 17 - Question 14

Where do the "finest art supplies " fit into the passage? While not mentioned explicitly, the author says in the last paragraph  "The largest brush, the brightest colors cannot even make an impression. " This happens, following the chain of the metaphor, because of a lack of reserves. Later, the reserves are defined: Proportion and Relation. Therefore, the best supplies aren't useful when these things are lacking. (B) paraphrases this perfectly.

Wrong answers:

(A): Out of Scope. The author mentions nothing about creative inspiration.

(C): Out of Scope. Another answer choice that introduces something (the viewer) that the author doesn't mention.

(D): Distortion. While the author does mention luck as what a general is left with when nothing is held in reserve, the author certainly doesn't think that painting has to be nothing more than luck.

CAT Mock Test - 17 - Question 15

Passage: In all battles two things are usually required of the Commander—in—Chief: to make a good plan for his army and to keep a strong reserve. Both of these are also obligatory for the painter. To make a plan, thorough reconnaissance of the country where the battle is to be fought is needed. Its fields, its mountains, its rivers, its bridges, its trees, its flowers, its atmosphere—all require and repay attentive observation from a special point of view. I think this is one of the chief delights that have come to me through painting. No doubt many people who are lovers of art have acquired it to a high degree without actually practicing.
But I expect that nothing will make one observe more quickly or more thoroughly than having to face the difficulty of representing the thing observed. And mind you, if you do observe accurately and with refinement, and if you do record what you have seen with tolerable correspondence, the result follows on the canvas with startling obedience. But in order to make his plan, the General must not only reconnoiter the battle—ground; he must also study the achievements of the great Captains of the past. He must bring the observations he has collected in the field into comparison with the treatment of similar incidents by famous chiefs. Considering that, the galleries of Europe take on a new—and to me at least a severely practical—interest. "This, then, is how —— painted a cataract.
Exactly, and there is that same light I noticed last week in the waterfall at ——. " And so on. You see the difficulty that baffled you yesterday; and you see how easily it has been overcome by a great or even by a skillful painter. Not only is your observation of Nature sensibly improved and developed, but also your comprehension of the masterpieces of art. But it is in the use and withholding of their reserves that the great commanders have generally excelled. After all, when once the last reserve has been thrown in, the commander's part is played.
If that does not win the battle, he has nothing else to give. Everything must be left to luck and to the fighting troops. But these last reserves, in the absence of high direction, are apt to get into sad confusion, all mixed together in a nasty mess, without order or plan—and consequently without effect. Mere masses count no more. The largest brush, the brightest colors cannot even make an impression. The pictorial battlefield becomes a sea of mud mercifully veiled by the fog of war. Even though the General plunges in himself and emerges bespattered, as he sometimes does, he will not retrieve the day. In painting, the reserves consist in Proportion or Relation.
And it is here that the art of the painter marches along the road which is traversed by all the greatest harmonies in thought. At one side of the palette there is white, at the other black; and neither is ever used ?neat.' Between these two rigid limits all the action must lie, all the power required must be generated. Black and white themselves placed in juxtaposition make no great impression; and yet they are the most that you can do in pure contrast.

Q. Below are listed various opinions that might possibly be ascribed to the author. Based on the passage, which of the following could most reasonably be attributed to the author?

Detailed Solution for CAT Mock Test - 17 - Question 15

The question asks what opinion could be ascribed to the author—what isn't stated, but can be inferred. Before looking at the answers, be sure you recall the author's opinions. While each of the three wrong answer choices is flawed, (D) has nothing specifically to do with painting, but does summarize how the author makes his argument. If the author didn't believe this, the argument in the passage could never have been made.

Wrong answers:

(A): Out of Scope. There's no contrast in the passage between talent and training. While the author seems to suggest that certain artistic abilities can be trained, there's nothing to indicate that he believes this at the expense of talent.

(B): Distortion. While the author does say in the first paragraph that fighting unsuccessfully is more exciting than winning, that doesn't necessarily mean that more is learned.

(C): Distortion. The author clearly believes that modern artists can learn from the masters, but there's nothing to suggest that he believes that the masters can't be equaled. The tone of this answer choice is far more severe and negative than the author's tone.

*Answer can only contain numeric values
CAT Mock Test - 17 - Question 16

The four sentences (labelled 1, 2, 3, and 4) below, when properly sequenced, would yield a coherent paragraph. Decide on the proper sequencing of the order of the sentences and key in the sequence of the four numbers as your answer:

1. However, such unbridled advancements have also escalated global issues like climate change, resource depletion, and significant socio-economic disparities.

2. The 21st century has been heralded as an era of technological marvels, transforming every facet of human life with unprecedented innovations.

3. These challenges necessitate a reconfiguration of global priorities towards sustainability and equitable progress, demanding collaborative, not isolated, innovation.

4. Consequently, the narrative of development has increasingly become a balancing act between embracing technological growth and acknowledging its consequential perils.


Detailed Solution for CAT Mock Test - 17 - Question 16

The sequence starts with the marvels of the current era (2), introduces the downside of these advancements (1), reflects on the resulting global narrative (4), and concludes with a call for a new direction (3).

*Answer can only contain numeric values
CAT Mock Test - 17 - Question 17

The four sentences (labelled 1, 2, 3, and 4) below, when properly sequenced, would yield a coherent paragraph. Decide on the proper sequencing of the order of the sentences and key in the sequence of the four numbers as your answer:

1. As these narratives unfold, they often obscure the complex realities, offering instead a simplified, often sensationalized, version of events.

2. In an increasingly interconnected world, global crises are a focal point of media coverage, shaping public opinion and international policies.

3. This trend raises critical questions about the responsibility and role of journalism in presenting nuanced portrayals of global issues.

4. Consequently, the power to shape perceptions rests not just with policymakers but significantly with those controlling the flow of information.


Detailed Solution for CAT Mock Test - 17 - Question 17

The sequence introduces the context of global crises and media coverage (2), discusses the effect of simplified narratives (1), reflects on the implications for perception shaping (4), and concludes with questions about journalistic responsibility (3).

*Answer can only contain numeric values
CAT Mock Test - 17 - Question 18

Directions: The four sentences (labelled 1, 2, 3 and 4) given in this question, when properly sequenced, form a coherent paragraph. Decide on the proper order for the sentences and key in this sequence of four numbers as your answer.

1. Knowing how to tell different hard drive noises apart, and learning what they might mean, can save your computer.

2. No PC is truly silent, but your computer shouldn't be louder than your lawnmower.

3. If your computer is making a clicking or grinding sound, or any sort of low-pitched buzzing noise, it could indicate a dying disk—do not ignore it.

4. If you have to turn up your music just to drown out the whirring or grinding noises your machine is making, you may want to look into that—it could be the first signal of some serious problems.


Detailed Solution for CAT Mock Test - 17 - Question 18

2 begins the sentence by introducing 'PC' as the subject of discussion. The sentence compares the PC with a lawnmower. Since a lawnmower's 'whirring' can only be drowned by music, 4, which suggests this idea, should follow 2. 3 is an example of our ability to differentiate among the different noises that a hard drive can make. Thus, 3 should follow 1 as it lists the three noises that you have to tell apart. Also, by not ignoring a low-pitched buzzing noise (as mentioned in 3) can help 'save your computer' (as mentioned in 1). So the correct order should be 2413.

*Answer can only contain numeric values
CAT Mock Test - 17 - Question 19

Directions: The four sentences (labelled 1, 2, 3 and 4) given in this question, when properly sequenced, form a coherent paragraph. Decide on the proper order for the sentences and key in this sequence of four numbers as your answer.

1. A good example of this principle is a floating ship.

2. The first person to realise a concept that related to buoyant forces was Archimedes.

3. This buoyant force is what allows any object to float on the surface of any liquid as long as the buoyant force is large enough to counter the weight of the object; otherwise, it will sink.

4. Archimedes' Principle states that the weight of the fluid that anybody displaces when submerged either partially or fully, is equal to the buoyant force exerted on the said body.


Detailed Solution for CAT Mock Test - 17 - Question 19

Statement 2 is a general statement and names the person who developed the concept so it should begin the paragraph. Then comes the name of the concept 'Archimedes' Principle' (in sentence 4) inspired from the name of the person who made it 'Archimedes' first introduced in Statement 2. So 4 should follow 2. This buoyant force in 3 was first introduced in 4 as the buoyant force, so 3 should follow 4. Sentence 1 states an example of this principle which is defined and explained in sentences 4 and 3, so it should come after them. So the correct order is 2431.

CAT Mock Test - 17 - Question 20

There is a sentence that is missing in the paragraph below. Look at the paragraph and decide in which blank (option 1, 2, 3, or 4) the following sentence would best fit.

Sentence: In this pursuit, the idea of establishing a human settlement on Mars has captivated scientists, policymakers, and entrepreneurs alike.

Paragraph: The final frontier, space, has always been a source of human fascination. Astronomers and space agencies have long directed their resources toward the exploration of this vast unknown. (1). The advancements in technology have enabled ambitious projects, such as probes sent to the farthest reaches of our solar system and telescopes peering into the depths of space and time. (2). Despite these technological leaps, space travel presents significant challenges, including the physiological effects on astronauts and the sustainability of life in hostile environments. (3). The prospect of interplanetary travel extends beyond scientific exploration, hinting at a potential future for humanity that could span multiple celestial bodies. (4).

Detailed Solution for CAT Mock Test - 17 - Question 20

The sentence introducing the concept of Mars colonization fits perfectly after the initial mention of space exploration's general goals, setting a specific context before discussing technological advancements in broader terms.

CAT Mock Test - 17 - Question 21

There is a sentence that is missing in the paragraph below. Look at the paragraph and decide in which blank (option 1, 2, 3, or 4) the following sentence would best fit.

Sentence: However, these advancements have also raised ethical concerns about data privacy and the potential loss of jobs due to automation.

Paragraph: Artificial Intelligence (AI) represents a revolutionary leap in technology and has permeated every aspect of human existence. From predictive algorithms used in medical diagnosis to smart assistants in homes, AI's applications seem limitless. (1). As AI continues to evolve, experts predict a future where machines can surpass human intelligence, a scenario often referred to as the "singularity." (2). The implications of such a future are hotly debated, with many advocating for stringent controls on AI development and use. (3). Despite the potential downsides, the pursuit of advanced AI could herald a new era of unprecedented technological growth and societal change. (4).

Detailed Solution for CAT Mock Test - 17 - Question 21

The missing sentence that discusses the negative aspects of AI advancements logically follows the initial introduction, offering a contrast that leads to further discussion on the implications of AI's evolution.

CAT Mock Test - 17 - Question 22

The most momentous development of our era, precisely, is the waning of the nation state: its inability to withstand countervailing 21st-century forces, and its calamitous loss of influence over human circumstance. National political authority is in decline, and, since we do not know any other sort, it feels like the end of the world. This is why a strange brand of apocalyptic nationalism is so widely in vogue. The current appeal of machismo as political style, the wall-building and xenophobia, the mythology and race theory, the fantastical promises of national restoration – these are not cures, but symptoms of what is slowly revealing itself to all: nation states everywhere are in an advanced state of political and moral decay from which they cannot individually extricate themselves.

Detailed Solution for CAT Mock Test - 17 - Question 22

This paragraph states that the nation state has weakened considerably in the 21st century. The decline of national political authority, in turn, has led to a strange brand of "apocalyptic nationalism" characterized by political machismo, xenophobia and the like.

Now, looking at the options, we see that option A sums up the key ideas of the paragraph well.

Option B, while true, does not touch upon apocalyptic nationalism, which is one of the main ideas in the given paragraph. The same is the case with option D. So, both options B and D are ruled out.

Option C is incorrect. The paragraph argues that the waning of the nation state has led to xenophobia and apocalyptic nationalism. Option C states the converse of this.

The question is "Choose the option that summarizes the paragraph best "

Hence, the answer is A

CAT Mock Test - 17 - Question 23

The passage given below is followed by four alternate summaries. Choose the option that best captures the essence of the passage.

For years, movies and television series like Crime Scene Investigation (CSI) paint an unrealistic picture of the “science of voices.” In the 1994 movie Clear and Present Danger an expert listens to a brief recorded utterance and declares that the speaker is “Cuban, aged 35 to 45, educated in the […] eastern United States.” The recording is then fed to a supercomputer that matches the voice to that of a suspect, concluding that the probability of correct identification is 90%. This sequence sums up a good number of misimpressions about forensic phonetics, which have led to errors in real-life justice. Indeed, that movie scene exemplifies the so-called “CSI effect”—the phenomenon in which judges hold unrealistic expectations of the capabilities of forensic science.

Detailed Solution for CAT Mock Test - 17 - Question 23

The main idea of the paragraph is conveyed in the last sentence: "Indeed, that movie scene exemplifies the so-called “CSI effect”—the phenomenon in which judges hold unrealistic expectations of the capabilities of forensic science." Option D rephrases this. Also note that all other option are specific to voice recognition. The paragraph is more general and talks of forensic science.

CAT Mock Test - 17 - Question 24

The passage given below is followed by four alternate summaries. Choose the option that best captures the essence of the passage.

As Soviet power declined, the world became to some extent multipolar, and Europe strove to define an independent identity. What a journey Europe has undertaken to reach this point. It had in every century changed its internal structure and invented new ways of thinking about the nature of international order. Now at the culmination of an era, Europe, in order to participate in it, felt obliged to set aside the political mechanisms through which it had conducted its affairs for three and a half centuries. Impelled also by the desire to cushion the emergent unification of Germany, the new European Union established a common currency in 2002 and a formal political structure in 2004. It proclaimed a Europe united, whole, and free, adjusting its differences by peaceful mechanisms.

Detailed Solution for CAT Mock Test - 17 - Question 24

The paragraph describes how Europe changed its internal structure and transformed itself into a united whole using peaceful mechanisms in the new multi-polar world. Option D captures all key ideas in the paragraph. The paragraph is specific to the time after Soviet decline and emergent unification of Germany-- a multi-polar world. Options A and C do not include this idea. The paragraph explains how Europe changed its internal structure by adjusting its differences by peaceful mechanisms. Option B does not include this.

CAT Mock Test - 17 - Question 25

Directions: Read the information given below and answer the question that follows.

Five contestants, Danny, Elizabeth, Ashok, Ramesh and Franklin, entered in a puzzle show which had four puzzles – First puzzle, Second puzzle, Third puzzle and Fourth puzzle. Each puzzle had three checkboxes, Checkbox-1, Checkbox-2 and Checkbox-3, and each contestant had to choose one of the three checkboxes as his/her answer for each. The five contestants were sitting in a line, one behind the other, facing the same direction, not necessarily in the same order as mentioned above. During the show, except for the contestant sitting at the beginning of the line, each contestant copied the answer to exactly one puzzle from the contestant immediately in front of him. Further, for any pair of contestants sitting immediately next to each other, exactly one answer was the same (i.e. the answer that was copied).

It is also known that:
(i) The contestant sitting at the last position, who was not Ashok, marked the answer as Checkbox-2 for Second puzzle and Franklin, who was not at the last position, did not mark the answer to Third puzzle as Checkbox-2.
(ii) Exactly two contestants marked the answer as Checkbox-2 for First puzzle and neither of the two was sitting at any of the ends.
(iii) Danny, who was sitting immediately in front of Ashok, marked the answer as Checkbox-3 for exactly two puzzles, while Ramesh marked the answer as Checkbox-3 only for First puzzle.
(iv) Each of the four contestants who copied the answer did so for a different puzzle and each contestant marked the answer as Checkbox-1 for at least one puzzle, as Checkbox-2 for at least one puzzle and as Checkbox-3 for at least one puzzle.
(v) The contestant sitting in the middle marked the answer to Second puzzle as Checkbox-3, which he did not copy.
(vi) The contestant sitting at the second position, who was not Ashok, marked the answer as Checkbox-2 for Third puzzle, which he copied.

Q. What is the answer marked for Fourth puzzle by the contestant sitting in the last position?

Detailed Solution for CAT Mock Test - 17 - Question 25

From (vi), the person sitting at the second position and the person sitting at the first position marked the answer to Third puzzle as Checkbox-2.
From (i), Franklin cannot be at the first or second positions because he did not mark the answer to Third puzzle as Checkbox-2.
According to (iii), Danny is sitting immediately in front of Ashok. From (vi), Ashok is not at the second position. From (i), Ashok was not at the last position. Hence, Danny and Ashok can be second and third OR third and fourth.

However, if Danny and Ashok are third and fourth, Franklin cannot be at the first, second, third or fourth position. From (i), Franklin cannot be at the last position as well.
Hence, this case is not possible. Therefore, Danny and Ashok are second and third. Franklin cannot be first or fifth. Hence, Franklin has to be fourth.

Danny would have marked the answer to Third puzzle as Checkbox-2. From (v), Ashok would have marked the answer to Second puzzle as Checkbox-3. From (iv), since Danny copied the answer to Third puzzle, none of the others could have copied the answer to the same puzzle. Since Ashok did not copy the answer to Third puzzle from Danny, his answer to Third puzzle must be Checkbox 1 or 3. From (v), Ashok did not copy the answer to Second Puzzle. Hence, Danny would have marked Checkbox-1 or Checkbox-2 for Second puzzle. From (iii), Danny would have marked Checkbox-3 for First puzzle and Fourth puzzle. Since each contestant marked each checkbox for at least one puzzle (from (iv)), Danny must have marked the answer as Checkbox-1 for Second puzzle. From (ii), two contestants marked the answer as Checkbox-2 for First puzzle. These two contestants are not at extreme ends. Since Danny (at second position) marked the answer as Checkbox-3 for First puzzle, Ashok and Franklin (at fourth position) would have marked the answer to First puzzle as Checkbox-2.

From (i), Ramesh marked the answer to First puzzle as Checkbox-3. If Ramesh was at the first position, his answer would be same as Danny's answer for this puzzle, which is not possible (since Danny copied Third puzzle from the person at the first position). Hence, Ramesh cannot be first and he will be fifth. The person at the first position will be Elizabeth.

Elizabeth must have marked the answer to First puzzle as Checkbox-1 (she could not mark Checkbox-2 since only two people marked that checkbox for First puzzle; she could not mark the answer as Checkbox-3 since Danny marked it). Elizabeth could mark the answer as Checkbox-3 only for Second puzzle. For Fourth puzzle, she could have marked as Checkbox-1 or Checkbox-2.

Ashok did not copy Second puzzle from Danny. Hence, he must have copied Fourth puzzle. Therefore, Ashok would have marked the answer to Fourth puzzle as Checkbox-3. From (iv), Ashok must mark Checkbox-1 as the answer for Third puzzle (since he did not mark Checkbox-1 for any other puzzle).

Since Franklin, Danny and Ashok copied First puzzle, Third puzzle and Fourth puzzle, respectively, Ramesh must have copied Second puzzle. From (i), Ramesh marked Checkbox-2 for Second puzzle. Hence, both Ramesh and Franklin would have marked Checkbox-2 for Second puzzle.
Since Franklin marked Checkbox-2 for First puzzle and Second puzzle. He must mark Checkbox-3 for Third puzzle and Checkbox-1 for Fourth puzzle (since Ashok marked Checkbox-1 and Checkbox-3 for Third puzzle and Fourth puzzle, respectively). Since Ramesh marked Checkbox-3 only for First puzzle, he must have marked Checkbox-2 for Fourth puzzle. For Third puzzle, he must have marked Checkbox-1 (from (iv)).

The following table presents the checkboxes marked by the four contestants:

*Answer can only contain numeric values
CAT Mock Test - 17 - Question 26

Directions: Read the information given below and answer the question that follows.

Five contestants, Danny, Elizabeth, Ashok, Ramesh and Franklin, entered in a puzzle show which had four puzzles – First puzzle, Second puzzle, Third puzzle and Fourth puzzle. Each puzzle had three checkboxes, Checkbox-1, Checkbox-2 and Checkbox-3, and each contestant had to choose one of the three checkboxes as his/her answer for each. The five contestants were sitting in a line, one behind the other, facing the same direction, not necessarily in the same order as mentioned above. During the show, except for the contestant sitting at the beginning of the line, each contestant copied the answer to exactly one puzzle from the contestant immediately in front of him. Further, for any pair of contestants sitting immediately next to each other, exactly one answer was the same (i.e. the answer that was copied).

It is also known that:
(i) The contestant sitting at the last position, who was not Ashok, marked the answer as Checkbox-2 for Second puzzle and Franklin, who was not at the last position, did not mark the answer to Third puzzle as Checkbox-2.
(ii) Exactly two contestants marked the answer as Checkbox-2 for First puzzle and neither of the two was sitting at any of the ends.
(iii) Danny, who was sitting immediately in front of Ashok, marked the answer as Checkbox-3 for exactly two puzzles, while Ramesh marked the answer as Checkbox-3 only for First puzzle.
(iv) Each of the four contestants who copied the answer did so for a different puzzle and each contestant marked the answer as Checkbox-1 for at least one puzzle, as Checkbox-2 for at least one puzzle and as Checkbox-3 for at least one puzzle.
(v) The contestant sitting in the middle marked the answer to Second puzzle as Checkbox-3, which he did not copy.
(vi) The contestant sitting at the second position, who was not Ashok, marked the answer as Checkbox-2 for Third puzzle, which he copied.

Q. If the positions in which the contestants are sitting are represented as 1 to 5, from first to last, for how many contestants is their position the same as the number of the puzzle for which they copied the answer?


Detailed Solution for CAT Mock Test - 17 - Question 26

From (vi), the person sitting at the second position and the person sitting at the first position marked the answer to Third puzzle as Checkbox-2.
From (i), Franklin cannot be at the first or second positions because he did not mark the answer to Third puzzle as Checkbox-2.
According to (iii), Danny is sitting immediately in front of Ashok. From (vi), Ashok is not at the second position. From (i), Ashok was not at the last position. Hence, Danny and Ashok can be second and third OR third and fourth.

However, if Danny and Ashok are third and fourth, Franklin cannot be at the first, second, third or fourth position. From (i), Franklin cannot be at the last position as well.
Hence, this case is not possible. Therefore, Danny and Ashok are second and third. Franklin cannot be first or fifth. Hence, Franklin has to be fourth.

Danny would have marked the answer to Third puzzle as Checkbox-2. From (v), Ashok would have marked the answer to Second puzzle as Checkbox-3. From (iv), since Danny copied the answer to Third puzzle, none of the others could have copied the answer to the same puzzle. Since Ashok did not copy the answer to Third puzzle from Danny, his answer to Third puzzle must be Checkbox 1 or 3. From (v), Ashok did not copy the answer to Second Puzzle. Hence, Danny would have marked Checkbox-1 or Checkbox-2 for Second puzzle. From (iii), Danny would have marked Checkbox-3 for First puzzle and Fourth puzzle. Since each contestant marked each checkbox for at least one puzzle (from (iv)), Danny must have marked the answer as Checkbox-1 for Second puzzle. From (ii), two contestants marked the answer as Checkbox-2 for First puzzle. These two contestants are not at extreme ends. Since Danny (at second position) marked the answer as Checkbox-3 for First puzzle, Ashok and Franklin (at fourth position) would have marked the answer to First puzzle as Checkbox-2.

From (i), Ramesh marked the answer to First puzzle as Checkbox-3. If Ramesh was at the first position, his answer would be same as Danny's answer for this puzzle, which is not possible (since Danny copied Third puzzle from the person at the first position). Hence, Ramesh cannot be first and he will be fifth. The person at the first position will be Elizabeth.

Elizabeth must have marked the answer to First puzzle as Checkbox-1 (she could not mark Checkbox-2 since only two people marked that checkbox for First puzzle; she could not mark the answer as Checkbox-3 since Danny marked it). Elizabeth could mark the answer as Checkbox-3 only for Second puzzle. For Fourth puzzle, she could have marked as Checkbox-1 or Checkbox-2.

Ashok did not copy Second puzzle from Danny. Hence, he must have copied Fourth puzzle. Therefore, Ashok would have marked the answer to Fourth puzzle as Checkbox-3. From (iv), Ashok must mark Checkbox-1 as the answer for Third puzzle (since he did not mark Checkbox-1 for any other puzzle).

Since Franklin, Danny and Ashok copied First puzzle, Third puzzle and Fourth puzzle, respectively, Ramesh must have copied Second puzzle. From (i), Ramesh marked Checkbox-2 for Second puzzle. Hence, both Ramesh and Franklin would have marked Checkbox-2 for Second puzzle.
Since Franklin marked Checkbox-2 for First puzzle and Second puzzle. He must mark Checkbox-3 for Third puzzle and Checkbox-1 for Fourth puzzle (since Ashok marked Checkbox-1 and Checkbox-3 for Third puzzle and Fourth puzzle, respectively). Since Ramesh marked Checkbox-3 only for First puzzle, he must have marked Checkbox-2 for Fourth puzzle. For Third puzzle, he must have marked Checkbox-1 (from (iv)).

The following table presents the checkboxes marked by the four contestants:

None of the contestants satisfies the given condition.

CAT Mock Test - 17 - Question 27

Directions: Read the information given below and answer the question that follows.

Five contestants, Danny, Elizabeth, Ashok, Ramesh and Franklin, entered in a puzzle show which had four puzzles – First puzzle, Second puzzle, Third puzzle and Fourth puzzle. Each puzzle had three checkboxes, Checkbox-1, Checkbox-2 and Checkbox-3, and each contestant had to choose one of the three checkboxes as his/her answer for each. The five contestants were sitting in a line, one behind the other, facing the same direction, not necessarily in the same order as mentioned above. During the show, except for the contestant sitting at the beginning of the line, each contestant copied the answer to exactly one puzzle from the contestant immediately in front of him. Further, for any pair of contestants sitting immediately next to each other, exactly one answer was the same (i.e. the answer that was copied).

It is also known that:
(i) The contestant sitting at the last position, who was not Ashok, marked the answer as Checkbox-2 for Second puzzle and Franklin, who was not at the last position, did not mark the answer to Third puzzle as Checkbox-2.
(ii) Exactly two contestants marked the answer as Checkbox-2 for First puzzle and neither of the two was sitting at any of the ends.
(iii) Danny, who was sitting immediately in front of Ashok, marked the answer as Checkbox-3 for exactly two puzzles, while Ramesh marked the answer as Checkbox-3 only for First puzzle.
(iv) Each of the four contestants who copied the answer did so for a different puzzle and each contestant marked the answer as Checkbox-1 for at least one puzzle, as Checkbox-2 for at least one puzzle and as Checkbox-3 for at least one puzzle.
(v) The contestant sitting in the middle marked the answer to Second puzzle as Checkbox-3, which he did not copy.
(vi) The contestant sitting at the second position, who was not Ashok, marked the answer as Checkbox-2 for Third puzzle, which he copied.

Q. For which of the following puzzles did both Ashok and the contestant sitting at the first position mark Checkbox-3 as their answer?

Detailed Solution for CAT Mock Test - 17 - Question 27

From (vi), the person sitting at the second position and the person sitting at the first position marked the answer to Third puzzle as Checkbox-2.
From (i), Franklin cannot be at the first or second positions because he did not mark the answer to Third puzzle as Checkbox-2.
According to (iii), Danny is sitting immediately in front of Ashok. From (vi), Ashok is not at the second position. From (i), Ashok was not at the last position. Hence, Danny and Ashok can be second and third OR third and fourth.

However, if Danny and Ashok are third and fourth, Franklin cannot be at the first, second, third or fourth position. From (i), Franklin cannot be at the last position as well.
Hence, this case is not possible. Therefore, Danny and Ashok are second and third. Franklin cannot be first or fifth. Hence, Franklin has to be fourth.

Danny would have marked the answer to Third puzzle as Checkbox-2. From (v), Ashok would have marked the answer to Second puzzle as Checkbox-3. From (iv), since Danny copied the answer to Third puzzle, none of the others could have copied the answer to the same puzzle. Since Ashok did not copy the answer to Third puzzle from Danny, his answer to Third puzzle must be Checkbox 1 or 3. From (v), Ashok did not copy the answer to Second Puzzle. Hence, Danny would have marked Checkbox-1 or Checkbox-2 for Second puzzle. From (iii), Danny would have marked Checkbox-3 for First puzzle and Fourth puzzle. Since each contestant marked each checkbox for at least one puzzle (from (iv)), Danny must have marked the answer as Checkbox-1 for Second puzzle. From (ii), two contestants marked the answer as Checkbox-2 for First puzzle. These two contestants are not at extreme ends. Since Danny (at second position) marked the answer as Checkbox-3 for First puzzle, Ashok and Franklin (at fourth position) would have marked the answer to First puzzle as Checkbox-2.

From (i), Ramesh marked the answer to First puzzle as Checkbox-3. If Ramesh was at the first position, his answer would be same as Danny's answer for this puzzle, which is not possible (since Danny copied Third puzzle from the person at the first position). Hence, Ramesh cannot be first and he will be fifth. The person at the first position will be Elizabeth.

Elizabeth must have marked the answer to First puzzle as Checkbox-1 (she could not mark Checkbox-2 since only two people marked that checkbox for First puzzle; she could not mark the answer as Checkbox-3 since Danny marked it). Elizabeth could mark the answer as Checkbox-3 only for Second puzzle. For Fourth puzzle, she could have marked as Checkbox-1 or Checkbox-2.

Ashok did not copy Second puzzle from Danny. Hence, he must have copied Fourth puzzle. Therefore, Ashok would have marked the answer to Fourth puzzle as Checkbox-3. From (iv), Ashok must mark Checkbox-1 as the answer for Third puzzle (since he did not mark Checkbox-1 for any other puzzle).

Since Franklin, Danny and Ashok copied First puzzle, Third puzzle and Fourth puzzle, respectively, Ramesh must have copied Second puzzle. From (i), Ramesh marked Checkbox-2 for Second puzzle. Hence, both Ramesh and Franklin would have marked Checkbox-2 for Second puzzle.
Since Franklin marked Checkbox-2 for First puzzle and Second puzzle. He must mark Checkbox-3 for Third puzzle and Checkbox-1 for Fourth puzzle (since Ashok marked Checkbox-1 and Checkbox-3 for Third puzzle and Fourth puzzle, respectively). Since Ramesh marked Checkbox-3 only for First puzzle, he must have marked Checkbox-2 for Fourth puzzle. For Third puzzle, he must have marked Checkbox-1 (from (iv)).

The following table presents the checkboxes marked by the four contestants:

Both Ashok and the person at the first position, Elizabeth, marked Checkbox-3 as their answer for Second puzzle.

CAT Mock Test - 17 - Question 28

Directions: Read the information given below and answer the question that follows.

Five contestants, Danny, Elizabeth, Ashok, Ramesh and Franklin, entered in a puzzle show which had four puzzles – First puzzle, Second puzzle, Third puzzle and Fourth puzzle. Each puzzle had three checkboxes, Checkbox-1, Checkbox-2 and Checkbox-3, and each contestant had to choose one of the three checkboxes as his/her answer for each. The five contestants were sitting in a line, one behind the other, facing the same direction, not necessarily in the same order as mentioned above. During the show, except for the contestant sitting at the beginning of the line, each contestant copied the answer to exactly one puzzle from the contestant immediately in front of him. Further, for any pair of contestants sitting immediately next to each other, exactly one answer was the same (i.e. the answer that was copied).

It is also known that:
(i) The contestant sitting at the last position, who was not Ashok, marked the answer as Checkbox-2 for Second puzzle and Franklin, who was not at the last position, did not mark the answer to Third puzzle as Checkbox-2.
(ii) Exactly two contestants marked the answer as Checkbox-2 for First puzzle and neither of the two was sitting at any of the ends.
(iii) Danny, who was sitting immediately in front of Ashok, marked the answer as Checkbox-3 for exactly two puzzles, while Ramesh marked the answer as Checkbox-3 only for First puzzle.
(iv) Each of the four contestants who copied the answer did so for a different puzzle and each contestant marked the answer as Checkbox-1 for at least one puzzle, as Checkbox-2 for at least one puzzle and as Checkbox-3 for at least one puzzle.
(v) The contestant sitting in the middle marked the answer to Second puzzle as Checkbox-3, which he did not copy.
(vi) The contestant sitting at the second position, who was not Ashok, marked the answer as Checkbox-2 for Third puzzle, which he copied.

Q. Which checkbox was marked by the maximum number of contestants for Fourth puzzle?

Detailed Solution for CAT Mock Test - 17 - Question 28

From (vi), the person sitting at the second position and the person sitting at the first position marked the answer to Third puzzle as Checkbox-2.
From (i), Franklin cannot be at the first or second positions because he did not mark the answer to Third puzzle as Checkbox-2.
According to (iii), Danny is sitting immediately in front of Ashok. From (vi), Ashok is not at the second position. From (i), Ashok was not at the last position. Hence, Danny and Ashok can be second and third OR third and fourth.

However, if Danny and Ashok are third and fourth, Franklin cannot be at the first, second, third or fourth position. From (i), Franklin cannot be at the last position as well.
Hence, this case is not possible. Therefore, Danny and Ashok are second and third. Franklin cannot be first or fifth. Hence, Franklin has to be fourth.

Danny would have marked the answer to Third puzzle as Checkbox-2. From (v), Ashok would have marked the answer to Second puzzle as Checkbox-3. From (iv), since Danny copied the answer to Third puzzle, none of the others could have copied the answer to the same puzzle. Since Ashok did not copy the answer to Third puzzle from Danny, his answer to Third puzzle must be Checkbox 1 or 3. From (v), Ashok did not copy the answer to Second Puzzle. Hence, Danny would have marked Checkbox-1 or Checkbox-2 for Second puzzle. From (iii), Danny would have marked Checkbox-3 for First puzzle and Fourth puzzle. Since each contestant marked each checkbox for at least one puzzle (from (iv)), Danny must have marked the answer as Checkbox-1 for Second puzzle. From (ii), two contestants marked the answer as Checkbox-2 for First puzzle. These two contestants are not at extreme ends. Since Danny (at second position) marked the answer as Checkbox-3 for First puzzle, Ashok and Franklin (at fourth position) would have marked the answer to First puzzle as Checkbox-2.

From (i), Ramesh marked the answer to First puzzle as Checkbox-3. If Ramesh was at the first position, his answer would be same as Danny's answer for this puzzle, which is not possible (since Danny copied Third puzzle from the person at the first position). Hence, Ramesh cannot be first and he will be fifth. The person at the first position will be Elizabeth.

Elizabeth must have marked the answer to First puzzle as Checkbox-1 (she could not mark Checkbox-2 since only two people marked that checkbox for First puzzle; she could not mark the answer as Checkbox-3 since Danny marked it). Elizabeth could mark the answer as Checkbox-3 only for Second puzzle. For Fourth puzzle, she could have marked as Checkbox-1 or Checkbox-2.

Ashok did not copy Second puzzle from Danny. Hence, he must have copied Fourth puzzle. Therefore, Ashok would have marked the answer to Fourth puzzle as Checkbox-3. From (iv), Ashok must mark Checkbox-1 as the answer for Third puzzle (since he did not mark Checkbox-1 for any other puzzle).

Since Franklin, Danny and Ashok copied First puzzle, Third puzzle and Fourth puzzle, respectively, Ramesh must have copied Second puzzle. From (i), Ramesh marked Checkbox-2 for Second puzzle. Hence, both Ramesh and Franklin would have marked Checkbox-2 for Second puzzle.
Since Franklin marked Checkbox-2 for First puzzle and Second puzzle. He must mark Checkbox-3 for Third puzzle and Checkbox-1 for Fourth puzzle (since Ashok marked Checkbox-1 and Checkbox-3 for Third puzzle and Fourth puzzle, respectively). Since Ramesh marked Checkbox-3 only for First puzzle, he must have marked Checkbox-2 for Fourth puzzle. For Third puzzle, he must have marked Checkbox-1 (from (iv)).

The following table presents the checkboxes marked by the four contestants:

Checkbox-3 was marked by two persons and one of Checkbox-1 and Checkbox-2 would be marked by two persons. Hence, more than one of the given checkboxes would have been marked by the maximum number of persons.

CAT Mock Test - 17 - Question 29

Directions: Read the information given below and answer the question that follows.

Five contestants, Danny, Elizabeth, Ashok, Ramesh and Franklin, entered in a puzzle show which had four puzzles – First puzzle, Second puzzle, Third puzzle and Fourth puzzle. Each puzzle had three checkboxes, Checkbox-1, Checkbox-2 and Checkbox-3, and each contestant had to choose one of the three checkboxes as his/her answer for each. The five contestants were sitting in a line, one behind the other, facing the same direction, not necessarily in the same order as mentioned above. During the show, except for the contestant sitting at the beginning of the line, each contestant copied the answer to exactly one puzzle from the contestant immediately in front of him. Further, for any pair of contestants sitting immediately next to each other, exactly one answer was the same (i.e. the answer that was copied).

It is also known that:
(i) The contestant sitting at the last position, who was not Ashok, marked the answer as Checkbox-2 for Second puzzle and Franklin, who was not at the last position, did not mark the answer to Third puzzle as Checkbox-2.
(ii) Exactly two contestants marked the answer as Checkbox-2 for First puzzle and neither of the two was sitting at any of the ends.
(iii) Danny, who was sitting immediately in front of Ashok, marked the answer as Checkbox-3 for exactly two puzzles, while Ramesh marked the answer as Checkbox-3 only for First puzzle.
(iv) Each of the four contestants who copied the answer did so for a different puzzle and each contestant marked the answer as Checkbox-1 for at least one puzzle, as Checkbox-2 for at least one puzzle and as Checkbox-3 for at least one puzzle.
(v) The contestant sitting in the middle marked the answer to Second puzzle as Checkbox-3, which he did not copy.
(vi) The contestant sitting at the second position, who was not Ashok, marked the answer as Checkbox-2 for Third puzzle, which he copied.

Q. Who among the following marked the answer to First puzzle as Checkbox-1?

Detailed Solution for CAT Mock Test - 17 - Question 29

From (vi), the person sitting at the second position and the person sitting at the first position marked the answer to Third puzzle as Checkbox-2.
From (i), Franklin cannot be at the first or second positions because he did not mark the answer to Third puzzle as Checkbox-2.
According to (iii), Danny is sitting immediately in front of Ashok. From (vi), Ashok is not at the second position. From (i), Ashok was not at the last position. Hence, Danny and Ashok can be second and third OR third and fourth.

However, if Danny and Ashok are third and fourth, Franklin cannot be at the first, second, third or fourth position. From (i), Franklin cannot be at the last position as well.
Hence, this case is not possible. Therefore, Danny and Ashok are second and third. Franklin cannot be first or fifth. Hence, Franklin has to be fourth.

Danny would have marked the answer to Third puzzle as Checkbox-2. From (v), Ashok would have marked the answer to Second puzzle as Checkbox-3. From (iv), since Danny copied the answer to Third puzzle, none of the others could have copied the answer to the same puzzle. Since Ashok did not copy the answer to Third puzzle from Danny, his answer to Third puzzle must be Checkbox 1 or 3. From (v), Ashok did not copy the answer to Second Puzzle. Hence, Danny would have marked Checkbox-1 or Checkbox-2 for Second puzzle. From (iii), Danny would have marked Checkbox-3 for First puzzle and Fourth puzzle. Since each contestant marked each checkbox for at least one puzzle (from (iv)), Danny must have marked the answer as Checkbox-1 for Second puzzle. From (ii), two contestants marked the answer as Checkbox-2 for First puzzle. These two contestants are not at extreme ends. Since Danny (at second position) marked the answer as Checkbox-3 for First puzzle, Ashok and Franklin (at fourth position) would have marked the answer to First puzzle as Checkbox-2.

From (i), Ramesh marked the answer to First puzzle as Checkbox-3. If Ramesh was at the first position, his answer would be same as Danny's answer for this puzzle, which is not possible (since Danny copied Third puzzle from the person at the first position). Hence, Ramesh cannot be first and he will be fifth. The person at the first position will be Elizabeth.

Elizabeth must have marked the answer to First puzzle as Checkbox-1 (she could not mark Checkbox-2 since only two people marked that checkbox for First puzzle; she could not mark the answer as Checkbox-3 since Danny marked it). Elizabeth could mark the answer as Checkbox-3 only for Second puzzle. For Fourth puzzle, she could have marked as Checkbox-1 or Checkbox-2.

Ashok did not copy Second puzzle from Danny. Hence, he must have copied Fourth puzzle. Therefore, Ashok would have marked the answer to Fourth puzzle as Checkbox-3. From (iv), Ashok must mark Checkbox-1 as the answer for Third puzzle (since he did not mark Checkbox-1 for any other puzzle).

Since Franklin, Danny and Ashok copied First puzzle, Third puzzle and Fourth puzzle, respectively, Ramesh must have copied Second puzzle. From (i), Ramesh marked Checkbox-2 for Second puzzle. Hence, both Ramesh and Franklin would have marked Checkbox-2 for Second puzzle.
Since Franklin marked Checkbox-2 for First puzzle and Second puzzle. He must mark Checkbox-3 for Third puzzle and Checkbox-1 for Fourth puzzle (since Ashok marked Checkbox-1 and Checkbox-3 for Third puzzle and Fourth puzzle, respectively). Since Ramesh marked Checkbox-3 only for First puzzle, he must have marked Checkbox-2 for Fourth puzzle. For Third puzzle, he must have marked Checkbox-1 (from (iv)).

The following table presents the checkboxes marked by the four contestants:

Elizabeth marked her answer as Checkbox-1 for First puzzle.

*Answer can only contain numeric values
CAT Mock Test - 17 - Question 30

Directions: Read the following and answer the question that follows.

Indra Co-operative Society began with a membership of three members on 1st January, 1900. The average age of this group was 18 2/3 years. Every year, a new member joined this society. The age of the individual person on 1st January of any year is an integer. The average age of the group as on 1st January, 1901 and on 1st January, 1902 was 20.75 years and 22.2 years, respectively. At any instance, the ages (in years) of the members of the group are all distinct. The eldest person in the group (of the five members) as on 1st January, 1902 was of 25 years of age. The age of any member cannot be less than 17 years on 1st January, 1900.

Q. If the eldest person among the ones, who first became members of the Indra Co-operative Society was only one year older than the other member, what was his age in years?


Detailed Solution for CAT Mock Test - 17 - Question 30

1. January, 1900:
Let the ages of the tlue members be ab,c as on 1. January, 1900, such that a<b<c


where d is the age of the new member as on 1. January, 1901
Thus, solving we get d= 24 years

Thus solving for e, we gete = 24 years
Since the eldstperson is 25 years old as our. Tan, 1902, and all individuals have distinct aged is the dder of all 
Also, none of all and c can be more than 21 years of age E,g, if any of ab and c is 22, thenhe Mlle of 24 y®s of age on 1. Ian 1902, and e is already 24 years old on 1. Jan 1902 (2 persons with same au is not possible) 
Thus a,b,c < 21 years 
The following combinations are possible for a,b,c
Ages as on 1st Plan, 1900 couldbe (17, 19, 20).07, 18, 21)
Since this is applicable in only 1 triplet (17,19,20), age of the eldest is 20 years.

View more questions
16 videos|27 docs|58 tests
Information about CAT Mock Test - 17 Page
In this test you can find the Exam questions for CAT Mock Test - 17 solved & explained in the simplest way possible. Besides giving Questions and answers for CAT Mock Test - 17, EduRev gives you an ample number of Online tests for practice

Top Courses for CAT

16 videos|27 docs|58 tests
Download as PDF

Top Courses for CAT